Wieso fällt durch den Widerstand keine Spannung ab?

Aufgabe - (Physik, Elektrik, Elektrotechnik)

5 Antworten

Vom Fragesteller als hilfreich ausgezeichnet

Weil auf beiden Seiten des Widerstandes die selbe Spannung anliegt -> d.h. am Widerstand selbst fallen 0 Volt ab, es fließt über ihn kein Strom. 

Im Grunde sind die obere und untere Reihe zwei Spannungsteiler, und da die linke Seite oben wie auch unten den selben Gesamtwiderstand hat wie die jeweils reichte Seite wird die Spannung einfach nur durch 2 geteilt. 

D.h. wenn du an A 10 Volt hast und an B 0 Volt liegt am mittleren Widerstand auf beiden Seiten 5 Volt an -> es fließt über diesen Widerstand kein Strom.

roromoloko 
Fragesteller
 09.01.2017, 16:58

Ich verstehe nicht ganz wieso oben und unten dieselbe Spannung herrscht, wenn oben doch ein Widerstand von 8/3R und unten nur von 2R herrscht? Oder teilt sich die Spannung unabhängig von den Widerständen auf?

0
HelftMir123  09.01.2017, 22:12
@roromoloko

weil der obere Zweig parallel zum unteren Zweig ist. Das heißt, dass logischer Weise ganz links in beiden Fällen die 10 Volt anliegen und ganz rechts die 0 Volt (ich gehe mal von meinem Beispiel A=10V und B=0V aus). 

Sieht man sich nun mal einen Zweig alleine an. Man nehme den unteren Zweig: links sind 10 V, dann kommt R und dann noch einmal R, dann 0 Volt. Da die beiden Widerstände in Reihe sind teilt sich die Spannung auf R (links) und R (rechts auf). Da sie außerdem gleich groß sind, fällt an beiden Widerständen genau die hälfte der Spannung ab, also 5 Volt -> in der Mitte zwischen R (links) und R (rechts) ist ein Potential von 5 V. 

Das Selbe gilt für den oberen Zweig. Wieder liegen ganz links 10 Volt an (da der obere Zweig ja parallel zum unteren ist und in einer Parallelschaltung die Spannungen gleich groß sind, nur die Ströme unterschiedlich), dann kommt R und R/3, dann wieder R und R/3, dann 0 Volt. R und R/3 links haben ja den selben Gesamtwert wie R und R/3 rechts. Auch hier teilt sich die Spannung also genau auf die Hälfte (nämlich 5 Volt). An R+R/3 (links) fällt somit ebenso 5 Volt ab wie an R+R/3 (rechts) -> wieder ist in der Mitte ein Potential von 5 Volt. 

Sieh dir noch einmal die Regeln in der Parallel- und Reihenschaltung an (wann ist der Strom gleich, wann die Spannung usw.) und überlege dann, welcher Widerstand in deiner Schaltung zu wem parallel/in Reihe ist. Außerdem hilft dir vielleicht eine kleine Recherche zur Brückenschaltung nach, genau das hast du hier nämlich :)

1
roromoloko 
Fragesteller
 09.01.2017, 22:51
@HelftMir123

Ach mensch, ich bin auch blöd.. Hab mir echt auf den Kopf geschlagen, dass ich nicht vergessen habe, dass das ja allgemein in einer Parallelschaltung gilt ...

Nur noch eine Frage.. Sie haben jetzt gerade die 0 Volt erklärt, da eine "Symmetrie" zwischen rechts und links herrscht, sodass man dazwischen auf 0 Volt kommt. Wäre die Symmetrie nicht vorhanden, würde doch dennoch 0 V dazwischen liegen, da ja bei einer Parallelschaltung die Spannungen gleich sind oder?

0
HelftMir123  09.01.2017, 23:05
@roromoloko

Die Symmetrie (ich denke mal, gemeint sind die Verhältnisse der Widerstandswerte, also jeweils links zu rechts wie 1 zu 1) bedingt ja die 0 Volt Spannungsdifferenz am mittleren Widerstand. Fehlt diese, dann liegt eine Spannung an und es fließt ein Strom. Ohne Symmetrie würde eine Seite des Widerstandes ja an einem höheren Potential liegen als die andere. 

Kann es sein, dass dein Denkfehler darin liegt, dass du die Parallelschaltung zwischen dem linken und rechten Teil vermutest? Das wäre falsch, der obere Zweig ist zum unteren parallel, der linke Teil ist jeweils in Reihe zum rechten Teil.

Das Ganze kann man sich auch wie 2 parallel geschaltete Potentiometer betrachten. Beide liegen auf der einen Seite an 10 Volt (Punkt A), auf der anderen an 0 Volt (Punkt B). Bringt man den Schleifer bei beiden Potentiometern in die Mitte, dann liegen beide Schleifer auf 5 Volt - egal ob der eine ein 10k-Ohm und der andere ein 2k-Ohm Poti ist oder beide 5k-Ohm Potis sind. Verbindet man nun die Schleifer mit einem Widerstand, so liegt an diesem keine Spannung (da beide Seiten an 5 Volt/den Schleifern) und es fließt kein Strom. 

Dreh mal im Kopf die Schaltung um 90° (so dass "A" oben ist und "B" unten), dann sieht man die Parallelschaltung deutlicher.

Danke fürs Sternchen :) 

0

Rechnerisch liegt an dem Widerstand keine Spannung an und daher wird er vernachlässigt.

Diese Aufgabe wurde so erstellt um die Lösung etwas komplizierter zu gestalten.

roromoloko 
Fragesteller
 09.01.2017, 16:59

Ich verstehe nicht ganz wieso oben und unten dieselbe Spannung herrscht, wenn oben doch ein Widerstand von 8/3R und unten nur von 2R herrscht? Oder teilt sich die Spannung unabhängig von den Widerständen auf?

0
ProfDrStrom  09.01.2017, 17:03
@roromoloko

Wenn Du die realen Zahlen hast, berechne doch die einzelnen Punkte mal. Dann wird es etwas klarer.

0
roromoloko 
Fragesteller
 09.01.2017, 17:15
@ProfDrStrom

Also ich weiß nicht was sich da groß ändern soll? 8/3 und 2 sind unterschiedliche Zahlen :D

0
ProfDrStrom  09.01.2017, 18:36
@roromoloko

Berechne die Spannung für den mittleren senkrecht stehenden Widerstand, dann wird es deutlich.

0

Der obere und der untere Spannungsteiler teilt jeweils im gleichen Verhältnis. Also ist am Widerstand oben und unter die gleiche Spannung.

Strom fliesst nur, wenn es einen Spannungsunterschied gibt - entspricht dem Ohmschen Gesetz: R = U/I oder I = U x R

roromoloko 
Fragesteller
 09.01.2017, 17:00

Ich verstehe nicht ganz wieso oben und unten dieselbe Spannung herrscht, wenn oben doch ein Widerstand von 8/3R und unten nur von 2R herrscht? Oder teilt sich die Spannung unabhängig von den Widerständen auf?

0
weckmannu  10.01.2017, 12:39
@roromoloko

Die Aufteilung hängt nur vom Verhältnis der Widerstände ab, nicht von ihrer Summe.

0

Die beiden Widerstandsreihen in der oberen und unteren Ebene halbieren die Spannung, d. h. am Widerstand, der beide Ebenen verbindet, befindet sich das gleiche Spannungspotenzial.

Sind beide Potenziale gleich, so ist der Potenzialunterschied = Null.

Ist jedoch der Potenzialunterschied = Null, kann kein Strom fließen, da Stromfluss das Vorhandensein eines Potenzialunterschieds voraussetzt.

Günter

roromoloko 
Fragesteller
 09.01.2017, 17:00

Ich verstehe nicht ganz wieso oben und unten dieselbe Spannung herrscht, wenn oben doch ein Widerstand von 8/3R und unten nur von 2R herrscht? Oder teilt sich die Spannung unabhängig von den Widerständen auf?

0
GuenterLeipzig  09.01.2017, 20:33
@roromoloko

Du hast oben 2 gleiche Widerstände 8/3 R und unten auch 2 gleiche Widerstände 2R, an denen die gleiche Spannung anliegt.

Wir sind und sicher eining, dass egal, wie hoch die jeweils gleichen Widerstände sind, sie die gesamte Spannung exakt in der Mitte teilen, solange oben zwei gleich große und unten 2 gleich große Widerstände zur Anwendung kommen.

Was in den beiden Zweigen unterschiedlich ist, ist der Strom.

Das aber ist für die Spannungsbetrachtung nicht wichtig.

Günter

0
roromoloko 
Fragesteller
 09.01.2017, 22:46
@GuenterLeipzig

Würde die Spannung unabhängig von den verwendeten Widerständen oben und unten gleich sein, weil sich die Stromstärke ändert? Ich verstehe leider immer noch nicht ganz wieso die Spannung oben und unten gleich sein soll..:(

0

An seinen enden herrscht das gleiche Potential, also keine Potentialdifferenz oder die Spannung von 0 V (alles gleiche Bedeutung). Wenn an ihm keine Spannung anliegt,  fließt kein Strom hindurch, so dass auch keine spannung abfällt: U = R* I . mit U=0V, also auch I=0A und umgekehrt.

roromoloko 
Fragesteller
 09.01.2017, 16:58

Ich verstehe nicht ganz wieso oben und unten dieselbe Spannung herrscht, wenn oben doch ein Widerstand von 8/3R und unten nur von 2R herrscht? Oder teilt sich die Spannung unabhängig von den Widerständen auf?

0
willi55  16.01.2017, 14:41
@roromoloko

Du hast eine komplexe Widerstandsschaltung. Vereinfache sie. Da sind in der oberen Reihe zwei widerstände in Reihe. Diese kannst du jeweils zusammenfassen zu einem Widerstand. 

ES gilt das Ohm'sche Gesetz. U = R*I. In den beiden horizontalen Schaltungsteilen liegen die Widerstände beide an den Punkten A und B (Spannungsquelle) an. Der Gesamtwiderstand eines horizontalen Abschnittes kann leicht ausgerechnet werden. Die daraus resultierende Stromstärke oben und auch unten fließt durch beide Widerstände (oben: die du vorher ausgerechnet hast (4/3R; unten: R) hindurch. Da sie gleich sind, ist der Spannungsabfall natürlich auch gleich. Das ist oben und unten der Fall. Damit liegt am mittleren Widerstand oben und unten die halbe Spannung der Stromquelle AB an. Also gibt es keine Spannungsdifferenz zwischen den beiden Anschlüssen des mittleren Widerstandes. Wo keine Spannungsdifferenz ist, kann kein Strom fließen. 

fertig.

0